LSAT and Law School Admissions Forum

Get expert LSAT preparation and law school admissions advice from PowerScore Test Preparation.

 SherryZ
  • Posts: 124
  • Joined: Oct 06, 2013
|
#12172
Hi, thank you for your time and help!!

Oct 1999 LSAT, Sec 1, LR Q22:

I chose C but the right answer is D. Could you explain why D is right and C is wrong? Also, I suck at solving JUSTIFY questions. Could you tell me how to approach this type of question?


---Sherry
 Jacques Lamothe
PowerScore Staff
  • PowerScore Staff
  • Posts: 50
  • Joined: Sep 24, 2013
|
#12201
Hey Sherry,

Justify questions are similar to Strengthen questions, in the sense that you are picking an answer choice with information to bolster the author's argument. The difference is that the answer to a Justify question strengthens the argument so well that now the conclusion MUST follow from the author's premises. The best way to approach these questions is to begin by identifying the conclusion. Then, when you are looking for the answer choice that contains the principle that justifies the conclusion, you know that the correct answer will contain information that links the premises to the conclusion in some way.

With that in mind, we can evaluate answer choices (C) and (D). The principle expressed by answer choice (C) claims that, in certain situations, a type of financial dependency should not be eliminated. However, the conclusion of the stimulus argument is not a claim that the government's efforts to reduce the liability of nuclear power plants is a bad idea. Instead, the conclusion is a claim about whether the public's fears of a nuclear accident are justified. So the principle expressed in (C) is related to one of the argument's stated premises about liability reduction, but the principle does not work with the premise to justify the conclusion.

On the other hand, answer choice (D) links the stated premises to the conclusion about justified public fears. We know from the premises that the government is trying to stop the nuclear industry from going bankrupt. We also know that the industry can go bankrupt only if injury claims against the industry can be sustained (i.e. if a nuclear accident hurts some people). The principle in (D) tells us that governments do not take action unless there is a real danger that a harm will occur. Since the government is trying to reduce nuclear power industry liability, and unlimited nuclear power industry liability can only hurt the industry if nuclear accidents are possible, we can combine that information with the principle in (D) to conclude that there is a real risk of a nuclear accident. If that is the case, then the public is justified in being worried about them. So by combining answer choice D with the premises of the stimulus, we can justify the author's conclusion that the public's fear is well founded.

I hope that helps! Let me know if i need to clarify any part of that explanation.

Jacques
 Nikki Siclunov
PowerScore Staff
  • PowerScore Staff
  • Posts: 1362
  • Joined: Aug 02, 2011
|
#12220
Hi Sherry,

Let me add a to Jacques' excellent explanation above.

Note that the question stem is a Strengthen—PR, not a Justify—PR because of the presence of the word “most” in the question stem, which weakens the force required of the correct answer. In a Strengthen—PR question, the correct answer will provide a premise that, when applied to the specific situation in the stimulus, helps support the conclusion. Since a principle is by definition a broad rule (usually conditional in nature), the presence of the Principle indicator serves to broaden the scope of the question, which requires a more abstract understanding of the underlying relationships in the argument.

The key to answering this question quickly and efficiently is first to simplify the stimulus, and then to formulate a suitable prephrase. The editorial argues that nuclear power plants are dangerous. Why? The government claims that we are safe from nuclear accidents, but wants to limit the nuclear industry's financial liability in case of an accident. Liability, however, is only a threat if someone is injured in a nuclear accident. So, the argument goes, we have reason to fear accidents in nuclear power plants.

As I mentioned in my explanation of your earlier question, this argument is quite weak: just because the government wants to limit liability in case of an accident doesn't mean that there is actually any danger of a nuclear accident. Answer choice (D) fixes that problem by establishing the following conditional relationship:

Act to prevent something :arrow: Real danger that it will occur

If we adopt this principle, and given the government's behavior as it is described in the editorial, we can conclude that there is a real and present danger of a nuclear accident. If answer choice (D) is true, the logical gap in the argument is fixed, and the conclusion - strengthened. This is why answer choice (D) is correct.

Hope this helps!
 Jerrymakehabit
  • Posts: 52
  • Joined: Jan 28, 2019
|
#63687
Nikki Siclunov wrote:Hi Sherry,

Let me add a to Jacques' excellent explanation above.

Note that the question stem is a Strengthen—PR, not a Justify—PR because of the presence of the word “most” in the question stem, which weakens the force required of the correct answer. In a Strengthen—PR question, the correct answer will provide a premise that, when applied to the specific situation in the stimulus, helps support the conclusion. Since a principle is by definition a broad rule (usually conditional in nature), the presence of the Principle indicator serves to broaden the scope of the question, which requires a more abstract understanding of the underlying relationships in the argument.

The key to answering this question quickly and efficiently is first to simplify the stimulus, and then to formulate a suitable prephrase. The editorial argues that nuclear power plants are dangerous. Why? The government claims that we are safe from nuclear accidents, but wants to limit the nuclear industry's financial liability in case of an accident. Liability, however, is only a threat if someone is injured in a nuclear accident. So, the argument goes, we have reason to fear accidents in nuclear power plants.

As I mentioned in my explanation of your earlier question, this argument is quite weak: just because the government wants to limit liability in case of an accident doesn't mean that there is actually any danger of a nuclear accident. Answer choice (D) fixes that problem by establishing the following conditional relationship:

Act to prevent something :arrow: Real danger that it will occur

If we adopt this principle, and given the government's behavior as it is described in the editorial, we can conclude that there is a real and present danger of a nuclear accident. If answer choice (D) is true, the logical gap in the argument is fixed, and the conclusion - strengthened. This is why answer choice (D) is correct.

Hope this helps!
Great explains Nikki! But I have a small area which is not quite clear to me. Can you please explain?

"Act to prevent situation --> Real danger of that situation"

How does the government' limiting the nuclear industry's liability in case of an accident equal to "Act to prevent something"? The stimulus says limiting its liability is for the benefit of protecting them from bankruptcy. How does the government' limiting the nuclear industry's liability prevent the nuclear accident?

Thanks
Jerry
 Jerrymakehabit
  • Posts: 52
  • Joined: Jan 28, 2019
|
#63697
I categorized all the elements in the stimulus as below and now it looks clear to me.

P1: Limit liability in case of nuclear accident = prevent (unlimited liability) = prevent bankruptcy due to nuclear accident
The three elements in P2 below are/is the "situation" in (D)
P2: Unlimited liability :arrow: injury claims are sustainable :arrow: injury results from a nuclear accident
C: Fear = real danger of the nuclear accident

(D): Act to prevent situation --> Real danger of that situation
P1 --> Fear (Real danger of injury results from a nuclear accident)

Let me know how you think.

Thanks
Jerry
 Adam Tyson
PowerScore Staff
  • PowerScore Staff
  • Posts: 5374
  • Joined: Apr 14, 2011
|
#63919
I'm not following your diagrams here, Jerrymakehabit. "Fear" is not a sufficient condition for anything in the stimulus, and the conclusion is not that the fear is of a nuclear accident. The conclusion is that the fear of nuclear accidents is justified, which means that nuclear accidents are possible.

The author establishes in the premises that the government admits the following:

IF unlimited liability is a threat, THEN injury claims against them can be sustained

and

IF injury claims can be sustained, THEN they must be caused by a nuclear accident

Those two ideas can be connected in a continuous conditional chain.

The author then claims that the fear of a nuclear accident is justified. Nuclear accidents are, in fact, possible. His evidence is that the government acted to prevent unlimited liability so the nuclear power companies don't go bankrupt. How does that prove that there is a threat of a nuclear accident? We need to connect the actions of the government - they took steps to limit liability - to that conditional chain, which starts with "IF there is actually a threat posed by unlimited liability. We need "IF you act to prevent unlimited liability, THEN unlimited liability really is a threat". That's answer D! You don't limit liability unless unlimited liability really is a problem!

I hope that helps clear things up.
User avatar
 nicizle
  • Posts: 40
  • Joined: Aug 07, 2024
|
#109934
Can someone explain why B isn't a good answer? I interpreted the nuclear industry as financially benefitting from the government limiting liability, thus strengthening the conclusion. Is B wrong because the argument focuses on the government limiting liability, rather than the finances themselves?
 Adam Tyson
PowerScore Staff
  • PowerScore Staff
  • Posts: 5374
  • Joined: Apr 14, 2011
|
#110616
B is incorrect because the kind of event we are talking about is a nuclear accident, and there is no indication in the stimulus that anyone will benefit, financially or otherwise, from a nuclear accident. Also, it's not clear who has control over the occurrence of those events. After all, they are accidents, and perhaps nobody can control whether an accident occurs or not?

Get the most out of your LSAT Prep Plus subscription.

Analyze and track your performance with our Testing and Analytics Package.